2017 AMC 12A Problems/Problem 10

Revision as of 15:52, 8 February 2017 by Speck (talk | contribs) (Created page with "==Problem== Chloé chooses a real number uniformly at random from the interval <math> [ 0,2017 ]</math>. Independently, Laurent chooses a real number uniformly at random from...")
(diff) ← Older revision | Latest revision (diff) | Newer revision → (diff)

Problem

Chloé chooses a real number uniformly at random from the interval $[ 0,2017 ]$. Independently, Laurent chooses a real number uniformly at random from the interval $[ 0 , 4034 ]$. What is the probability that Laurent's number is greater than Chloe's number?

$\textbf{(A)}\ \dfrac{1}{2} \qquad\textbf{(B)}\ \dfrac{2}{3} \qquad\textbf{(C)}\ \dfrac{3}{4} \qquad\textbf{(D)}\ \dfrac{5}{6} \qquad\textbf{(E)}\ \dfrac{7}{8}$